Chapter 284. Quiz for Document 18-3

284.1 Section Title

true
true
Chapter 18: European Power and Expansion 1500–1750
Quiz for Document 18-3: The English Parliament Asserts Its Authority

Select the best answer for each question. Click the “Submit” button for each question to turn in your work. Click the "Previous" button to navigate back if you need to view the textbook.

Question 284.1

Correct. The answer is c. With memories of the English Civil War still fresh, the Bill of Rights prohibited kings from raising a standing army without the consent of Parliament.
Incorrect. The answer is c. With memories of the English Civil War still fresh, the Bill of Rights prohibited kings from raising a standing army without the consent of Parliament.
1C92/nrbcEP6Na3pz7D8l4WMcy84Z7XRKs6e58a1COmJnOEvmjWDwOMd4YSRyvPPZuQIn+tjuukUT8bVqEBP8rnK0O0m/e53iObQh/28Xa+HFLywtGQnu1JADYH+cg8CA5oSOA4jH8D3t/25q/boygCS22BSe3LuhWYbPukXVSzREl+3g0/qIDVcYPFHag2UGKEFrUxH1zaC+UwKbYpfay+M3XuoiQTeq2LUjbUiU4hsOsWvR9ilcdMFJfL+Y9GkPA0hAzgyrwUYqeZZ5M2gmkvM37Pn0NrZNG55MK+gEBNXpJipcgXfWjHHMJjGK/SVLp8uB2OenuDI4Yjh8vYb1sfzjpxEHuT7DfuhmmQYY3qnYJ2x7wNar9hGQ9mwkmhmJsz+ZVEADQ9gTiZImOKp6LIRnwa9Ew1izOalZ6CyvZ1352uNXXA7R/U72zE=

Question 284.2

Correct. The answer is a. One of the charges laid against King James II was that he tried to subvert Protestantism.
Incorrect. The answer is a. One of the charges laid against King James II was that he tried to subvert Protestantism.
Yro2Idnrci06A2U9VJdoXowcFjMnodBa8Tc2C0hhw2wyXTBQwyvMtvNehu56fB+Zy/MwmyZxiZXy0Kbh9OIXbA9canDFCeExgegttqeRD/tPhL+7VHpjaLIaNJ6tYt1IwGLfJEPG2JFFCAMjDuSqQRGNMVWsO2q9HT5WVw1gyQSxJJ28EG3Zp5M8eBs89yaq0zbo/uC+wcoew0KrmL9utbXN4l5QctqcONeP494iUOOz4vOKNOLnxyO5TDHRBSqgTF6vFD6GLTtYllghpZbMOfot2hUHtb36fVWJC+qJcxhWYQ5pHhBTjx16/NWd8hfU9Sn/1rbQ63uhmm70Y+tRVpFn1OAGLrkLLwMFpNugTfn9qG9NF+5R8SX9a1duY3sbnUuKs2aXEGoA+GE3gMaqNqf+l1BxvlMQG3OFwA==

Question 284.3

Correct. The answer is a. The Bill of Rights protected the power of Parliament in several ways, including protecting the freedom to debate in Parliament and dictating that Parliament elections should be free.
Incorrect. The answer is a. The Bill of Rights protected the power of Parliament in several ways, including protecting the freedom to debate in Parliament and dictating that Parliament elections should be free.
bktF6GyursQMhrG4A94VMa+UyN29EtL8PQ1GgBXHc7Xp4Ltu2flF8r8+CiJl65pP19ueIzKLZ+k+2P295dU481G4bRWgPwP1Oh14MM3bsMDZtLqoVMU9vjc6qlXwkXhNhsRd9zDMNWXU26B2RDeeM5pwwR3vw0G0HiCCn4kwJB3UAgdhR6LhAEDMswzkpomZZDug32WoX3arpRM2fJ7FLblTzIdx6GOkOWvzjFPv2YS3L85yCTigscRaEykl91M+SmOcL4Cj9hryB1R6Qy9wKmWbpuns3ZUdOPX8WXFAsQ4uCRKpQ8+0P1nFVNXDfBw5xyEpLUI5dFTdg2ONFAkru+5kMR1BC8jsWCLDTxu1Z4D21QbBxcSOc353LNDj/J9fTaCIeIUSGDvIDgtWNtNtf8eg6QoS3a/b9mFOIP3+g7Yx2INDWw5ZM8sdw9URtZEX0umyDNWNssvICek3

Question 284.4

Correct. The answer is b. The king could raise taxes only when he had the permission of Parliament.
Incorrect. The answer is b. The king could raise taxes only when he had the permission of Parliament.
KJpQloAJBClaKlBP0t89Sfr+k3HGf43v2BrYo7fHmYXvCc847JPVB9CrTj0iuytzGbe/fgfvGBhTtrW0eBYb+1ePMm8IFMkzYyHoQjJgyF5mlF1D/25smZpgqjXOWi0zSUsZqUD1JSvg8lAc/ekdLIokim/h6IGzRJkGwtWeq7UFSgRqhKI+++Xb9G7UKmsWjIk60gmjk0HiEy2g/nCXBgANDZhBTkQc1F3Est0mPZiAqfOgSio29RH71tC06FK4em0WG3AVZtBDEoZRSbx/A6hJRb0y6dF61E5ikPfChll58h/q4i/bsSXgUaBBsIkRSwAfPcrDy+hk35Z6UaxBCcsBt78=